1answer.
Ask question
Login Signup
Ask question
All categories
  • English
  • Mathematics
  • Social Studies
  • Business
  • History
  • Health
  • Geography
  • Biology
  • Physics
  • Chemistry
  • Computers and Technology
  • Arts
  • World Languages
  • Spanish
  • French
  • German
  • Advanced Placement (AP)
  • SAT
  • Medicine
  • Law
  • Engineering
Dmitriy789 [7]
3 years ago
14

$1000 is invested in an account at interest rate r=0.045, compounded continuously. Find the time required for the amount to doub

le
Mathematics
1 answer:
natulia [17]3 years ago
3 0

Answer:

The formula you'll need is:

Years = ln (total / principal) / rate

ln means natural log

Years = ln (2,000 / 1,000) / .045

Years = ln (2) / .045

Years = 0.69314718056 / .045

Years = 15.4033

Step-by-step explanation:

You might be interested in
Enter your answers in the boxes to correctly complete this statement.
harina [27]

Answer:

349800

4

Step-by-step explanation:

First Part:

The question tells you to multiply by 10 to the power of 4.

This implies that you can move the decimal point 4 numbers to the right

When you move the decimal two numbers to the right you get 3498

Then you got to move it two more numbers to the right

3498 can also be written as 3498.0000000000 (infinite amount of zeros)

So moving the decimal two more places you get 349800

Second Part:

10^4 = 10 * 10 * 10 * 10

= 10000

Count the zeros you get 4.

3 0
3 years ago
A square has vertices T(1,2), U(4,2), V(4,5), W(1,5). If the square is rotated clockwise 90 degrees around (0,0), what are the c
erastovalidia [21]
<span>(5,-1) The key to this problem is ignoring all the information that you don't need. You really don't care about the coordinates for T, U, or V. Just W is all that matters. So make a graph, label the X and Y axis, and put a point at (1,5). Now simply turn the entire paper clockwise 90 degrees. If you do this, you'll see that what was the X axis is now the Y axis. And what was the Y axis is now the X axis. Label the axis again, and read the location of the point you marked. If you do so, you'll see that you have to go 5 spaces along the X axis, hence the 5. And you'll go 1 space down on the Y axis, hence the -1. So the coordinates of W(1,5) rotated 90 degrees clockwise around (0,0) changes to (5,-1).</span>
6 0
3 years ago
The sides of the base of a right square pyramid are 5 centimeters, and the slant height is 8 centimeters. If the sides of the ba
maxonik [38]

Answer:

A

Step-by-step explanation:

3 0
3 years ago
Read 2 more answers
Pls help me with this​
Free_Kalibri [48]

Answer:

\boxed{\sf \frac{15}{22} }

Step-by-step explanation:

The radius of the circle is 7 cm.

The two legs of the triangles are 7cm as well.

Area of a trinagle is ( base × height )/2.

\frac{7 \times 7}{2} =24.5

Multiply the value by 2 since there are two triangles.

24.5 \times 2 = 49

Calculate the area of the circle.

\pi r^2

The radius is given.

\pi (7)^2

Take \pi as \frac{22}{7}

49(\frac{22}{7} )

=154

Subtract the area of the two triangles from the area of the whole circle.

154-49=105

The area of the shaded part is 105 cm². The total area of the shape is 154cm².

\frac{105}{154} =\frac{15}{22}

5 0
3 years ago
The edge of a cube is 4 inches. The area of a cross section perpendicular to one face is ___square inches, and its perimeter is_
diamong [38]
Try 8 and 12.......................................

3 0
3 years ago
Read 2 more answers
Other questions:
  • Someone please help me out
    12·1 answer
  • 4. PLEASE HELP ME WITH THIS MATH HOMEWORK
    14·1 answer
  • 3x+7=13 what's the answer?
    7·2 answers
  • 2/4 times 1/2 +(-1/8)
    6·1 answer
  • Find the total surface area. A. 2,016 km² B. 6,156 km² C. 1,368 km² D. 165 km²
    12·1 answer
  • based on the given information which theorem could be used to prove that angle PQR is greater than or equal to angle TSR? QR is
    12·1 answer
  • Find the area of the hexagen whose Vertices taken in order
    10·1 answer
  • The formula for the cost of buying a car is: Cost= 12 x monthly payment + deposit Find the cost of a car when the monthly paymen
    7·1 answer
  • What is the result when the number 84 is decreased by 50%?
    5·2 answers
  • Is 91,068,929 divisible by 4?<br><br> YES OR NO
    11·2 answers
Add answer
Login
Not registered? Fast signup
Signup
Login Signup
Ask question!